LSAT and Law School Admissions Forum

Get expert LSAT preparation and law school admissions advice from PowerScore Test Preparation.

 biskam
  • Posts: 124
  • Joined: Aug 18, 2017
|
#40684
Is A incorrect because we can't know this for sure? As in, "no gas" is extreme
I ruled out B bc I thought the choice should instead read "...sudden increases in the proportion of liquid to gas" because according to line 25 the surges in gas cause loss of pressure. So why is B right?

Thank you!
 James Finch
PowerScore Staff
  • PowerScore Staff
  • Posts: 943
  • Joined: Sep 06, 2017
|
#42247
Hi Biskam,

Yes, answer choice (A) is incorrect because the "no gas" qualifier isn't supported by the text. In fact, on line 24 we have an example of "60 percent liquid" (with the remaining 40 percent presumably gas) as a mixture at which the pumps can effectively pump the crude oil.

Answer choice (B) is correct because of those same lines 23-29: it is the "surge in gas content" that renders the pumps unable to work properly, so we can infer that if there are no sudden spikes in the percentage of gas-to-liquid in the crude oil, the pumps will work more efficiently than if they have to deal with these surges.

Hope that clears things up!

Get the most out of your LSAT Prep Plus subscription.

Analyze and track your performance with our Testing and Analytics Package.